There is only s̶i̶x̶t̶y one place he can be












17












$begingroup$


This puzzle is not mine, although I don't have any reference as to where it originated from. I discovered it when I was a student, some fifteen years ago. It has been among my favourite chess puzzles since then. Could be very well known or a duplicate, but I couldn't find it in the database.



The following position is legal, except that the White King (and the White King only) is not displayed. Find it.



                  










share|improve this question









$endgroup$








  • 2




    $begingroup$
    It appears this problem may have originated from The Chess Mysteries of the Arabian Knights by Raymond Smullyan.
    $endgroup$
    – noedne
    14 hours ago
















17












$begingroup$


This puzzle is not mine, although I don't have any reference as to where it originated from. I discovered it when I was a student, some fifteen years ago. It has been among my favourite chess puzzles since then. Could be very well known or a duplicate, but I couldn't find it in the database.



The following position is legal, except that the White King (and the White King only) is not displayed. Find it.



                  










share|improve this question









$endgroup$








  • 2




    $begingroup$
    It appears this problem may have originated from The Chess Mysteries of the Arabian Knights by Raymond Smullyan.
    $endgroup$
    – noedne
    14 hours ago














17












17








17





$begingroup$


This puzzle is not mine, although I don't have any reference as to where it originated from. I discovered it when I was a student, some fifteen years ago. It has been among my favourite chess puzzles since then. Could be very well known or a duplicate, but I couldn't find it in the database.



The following position is legal, except that the White King (and the White King only) is not displayed. Find it.



                  










share|improve this question









$endgroup$




This puzzle is not mine, although I don't have any reference as to where it originated from. I discovered it when I was a student, some fifteen years ago. It has been among my favourite chess puzzles since then. Could be very well known or a duplicate, but I couldn't find it in the database.



The following position is legal, except that the White King (and the White King only) is not displayed. Find it.



                  







chess






share|improve this question













share|improve this question











share|improve this question




share|improve this question










asked 15 hours ago









Arnaud MortierArnaud Mortier

1,624522




1,624522








  • 2




    $begingroup$
    It appears this problem may have originated from The Chess Mysteries of the Arabian Knights by Raymond Smullyan.
    $endgroup$
    – noedne
    14 hours ago














  • 2




    $begingroup$
    It appears this problem may have originated from The Chess Mysteries of the Arabian Knights by Raymond Smullyan.
    $endgroup$
    – noedne
    14 hours ago








2




2




$begingroup$
It appears this problem may have originated from The Chess Mysteries of the Arabian Knights by Raymond Smullyan.
$endgroup$
– noedne
14 hours ago




$begingroup$
It appears this problem may have originated from The Chess Mysteries of the Arabian Knights by Raymond Smullyan.
$endgroup$
– noedne
14 hours ago










2 Answers
2






active

oldest

votes


















15












$begingroup$

The white king must be on square




c3




Explanation:




First, we determine which side is to move. Unless the white king blocks the a4-d1 diagonal, the black king is in check. The king can't be on c2 since it would attack the black king; on b3, White would be to move but there's no way this double-check could have happened in a real game; there's no way to clear both the b3-d5 diagonal and the b3-b5 file with one move from the black rook or the black bishop. Black simply couldn't have played Bb4-d5 or Rc4-b5. So the black king is in check and White made the last move.




So, what was the last move?




The bishop can't have moved along the d1-a4 diagonal, because it would give check on c2 and d3 as well. So it must be a discovered check, caused by a move from the white king, from b3 to another square.




But ...




doesn't this lead to the same problem as before? On b3, the White king is still in double check. No, because there is a move which simultaneously clears the b3-d5 diagonal and the b3-b5 file, but it requires extra pieces: an en-passant capture from b4 to c3, capturing a pawn on c4. Since we have an extra (half-)move compared to the situation above, we can use that move to get rid of the black pawn.




So,




if White played 1. c2-c4, Black can play 1... b4xc3 e.p., White responds with 2. Kxc3 and we're in the diagram in the question with the White king on c3.







share|improve this answer











$endgroup$













  • $begingroup$
    ... but it doesn't include an explanation why this is the only possible solution (which took me a while to write down...)
    $endgroup$
    – Glorfindel
    14 hours ago






  • 1




    $begingroup$
    @ArnaudMortier Glorfindel's answer eliminates other possibilities.
    $endgroup$
    – noedne
    14 hours ago



















11












$begingroup$


The last move in this position was Bd5+. White blocked the check with c4, black gave double check by capturing en passant bxc, and the king took the pawn on c3.
chessboard







share|improve this answer









$endgroup$













  • $begingroup$
    Um... to me, it looks like rot13(gur cnja vf ba p2)... :P
    $endgroup$
    – EKons
    14 hours ago










  • $begingroup$
    @EKons I'm not sure I understand what you mean.
    $endgroup$
    – noedne
    14 hours ago










  • $begingroup$
    I've ROT13-ed part of the comment so that it doesn't appear as an immediate spoiler. Check the image you've posted.
    $endgroup$
    – EKons
    14 hours ago












  • $begingroup$
    @EKons I don't see what the problem is.
    $endgroup$
    – noedne
    14 hours ago










  • $begingroup$
    That was super quick. Did you know the problem before? Do you know where it is from?
    $endgroup$
    – Arnaud Mortier
    14 hours ago











Your Answer





StackExchange.ifUsing("editor", function () {
return StackExchange.using("mathjaxEditing", function () {
StackExchange.MarkdownEditor.creationCallbacks.add(function (editor, postfix) {
StackExchange.mathjaxEditing.prepareWmdForMathJax(editor, postfix, [["$", "$"], ["\\(","\\)"]]);
});
});
}, "mathjax-editing");

StackExchange.ready(function() {
var channelOptions = {
tags: "".split(" "),
id: "559"
};
initTagRenderer("".split(" "), "".split(" "), channelOptions);

StackExchange.using("externalEditor", function() {
// Have to fire editor after snippets, if snippets enabled
if (StackExchange.settings.snippets.snippetsEnabled) {
StackExchange.using("snippets", function() {
createEditor();
});
}
else {
createEditor();
}
});

function createEditor() {
StackExchange.prepareEditor({
heartbeatType: 'answer',
autoActivateHeartbeat: false,
convertImagesToLinks: false,
noModals: true,
showLowRepImageUploadWarning: true,
reputationToPostImages: null,
bindNavPrevention: true,
postfix: "",
imageUploader: {
brandingHtml: "Powered by u003ca class="icon-imgur-white" href="https://imgur.com/"u003eu003c/au003e",
contentPolicyHtml: "User contributions licensed under u003ca href="https://creativecommons.org/licenses/by-sa/3.0/"u003ecc by-sa 3.0 with attribution requiredu003c/au003e u003ca href="https://stackoverflow.com/legal/content-policy"u003e(content policy)u003c/au003e",
allowUrls: true
},
noCode: true, onDemand: true,
discardSelector: ".discard-answer"
,immediatelyShowMarkdownHelp:true
});


}
});














draft saved

draft discarded


















StackExchange.ready(
function () {
StackExchange.openid.initPostLogin('.new-post-login', 'https%3a%2f%2fpuzzling.stackexchange.com%2fquestions%2f81057%2fthere-is-only-s%25cc%25b6i%25cc%25b6x%25cc%25b6t%25cc%25b6y-one-place-he-can-be%23new-answer', 'question_page');
}
);

Post as a guest















Required, but never shown

























2 Answers
2






active

oldest

votes








2 Answers
2






active

oldest

votes









active

oldest

votes






active

oldest

votes









15












$begingroup$

The white king must be on square




c3




Explanation:




First, we determine which side is to move. Unless the white king blocks the a4-d1 diagonal, the black king is in check. The king can't be on c2 since it would attack the black king; on b3, White would be to move but there's no way this double-check could have happened in a real game; there's no way to clear both the b3-d5 diagonal and the b3-b5 file with one move from the black rook or the black bishop. Black simply couldn't have played Bb4-d5 or Rc4-b5. So the black king is in check and White made the last move.




So, what was the last move?




The bishop can't have moved along the d1-a4 diagonal, because it would give check on c2 and d3 as well. So it must be a discovered check, caused by a move from the white king, from b3 to another square.




But ...




doesn't this lead to the same problem as before? On b3, the White king is still in double check. No, because there is a move which simultaneously clears the b3-d5 diagonal and the b3-b5 file, but it requires extra pieces: an en-passant capture from b4 to c3, capturing a pawn on c4. Since we have an extra (half-)move compared to the situation above, we can use that move to get rid of the black pawn.




So,




if White played 1. c2-c4, Black can play 1... b4xc3 e.p., White responds with 2. Kxc3 and we're in the diagram in the question with the White king on c3.







share|improve this answer











$endgroup$













  • $begingroup$
    ... but it doesn't include an explanation why this is the only possible solution (which took me a while to write down...)
    $endgroup$
    – Glorfindel
    14 hours ago






  • 1




    $begingroup$
    @ArnaudMortier Glorfindel's answer eliminates other possibilities.
    $endgroup$
    – noedne
    14 hours ago
















15












$begingroup$

The white king must be on square




c3




Explanation:




First, we determine which side is to move. Unless the white king blocks the a4-d1 diagonal, the black king is in check. The king can't be on c2 since it would attack the black king; on b3, White would be to move but there's no way this double-check could have happened in a real game; there's no way to clear both the b3-d5 diagonal and the b3-b5 file with one move from the black rook or the black bishop. Black simply couldn't have played Bb4-d5 or Rc4-b5. So the black king is in check and White made the last move.




So, what was the last move?




The bishop can't have moved along the d1-a4 diagonal, because it would give check on c2 and d3 as well. So it must be a discovered check, caused by a move from the white king, from b3 to another square.




But ...




doesn't this lead to the same problem as before? On b3, the White king is still in double check. No, because there is a move which simultaneously clears the b3-d5 diagonal and the b3-b5 file, but it requires extra pieces: an en-passant capture from b4 to c3, capturing a pawn on c4. Since we have an extra (half-)move compared to the situation above, we can use that move to get rid of the black pawn.




So,




if White played 1. c2-c4, Black can play 1... b4xc3 e.p., White responds with 2. Kxc3 and we're in the diagram in the question with the White king on c3.







share|improve this answer











$endgroup$













  • $begingroup$
    ... but it doesn't include an explanation why this is the only possible solution (which took me a while to write down...)
    $endgroup$
    – Glorfindel
    14 hours ago






  • 1




    $begingroup$
    @ArnaudMortier Glorfindel's answer eliminates other possibilities.
    $endgroup$
    – noedne
    14 hours ago














15












15








15





$begingroup$

The white king must be on square




c3




Explanation:




First, we determine which side is to move. Unless the white king blocks the a4-d1 diagonal, the black king is in check. The king can't be on c2 since it would attack the black king; on b3, White would be to move but there's no way this double-check could have happened in a real game; there's no way to clear both the b3-d5 diagonal and the b3-b5 file with one move from the black rook or the black bishop. Black simply couldn't have played Bb4-d5 or Rc4-b5. So the black king is in check and White made the last move.




So, what was the last move?




The bishop can't have moved along the d1-a4 diagonal, because it would give check on c2 and d3 as well. So it must be a discovered check, caused by a move from the white king, from b3 to another square.




But ...




doesn't this lead to the same problem as before? On b3, the White king is still in double check. No, because there is a move which simultaneously clears the b3-d5 diagonal and the b3-b5 file, but it requires extra pieces: an en-passant capture from b4 to c3, capturing a pawn on c4. Since we have an extra (half-)move compared to the situation above, we can use that move to get rid of the black pawn.




So,




if White played 1. c2-c4, Black can play 1... b4xc3 e.p., White responds with 2. Kxc3 and we're in the diagram in the question with the White king on c3.







share|improve this answer











$endgroup$



The white king must be on square




c3




Explanation:




First, we determine which side is to move. Unless the white king blocks the a4-d1 diagonal, the black king is in check. The king can't be on c2 since it would attack the black king; on b3, White would be to move but there's no way this double-check could have happened in a real game; there's no way to clear both the b3-d5 diagonal and the b3-b5 file with one move from the black rook or the black bishop. Black simply couldn't have played Bb4-d5 or Rc4-b5. So the black king is in check and White made the last move.




So, what was the last move?




The bishop can't have moved along the d1-a4 diagonal, because it would give check on c2 and d3 as well. So it must be a discovered check, caused by a move from the white king, from b3 to another square.




But ...




doesn't this lead to the same problem as before? On b3, the White king is still in double check. No, because there is a move which simultaneously clears the b3-d5 diagonal and the b3-b5 file, but it requires extra pieces: an en-passant capture from b4 to c3, capturing a pawn on c4. Since we have an extra (half-)move compared to the situation above, we can use that move to get rid of the black pawn.




So,




if White played 1. c2-c4, Black can play 1... b4xc3 e.p., White responds with 2. Kxc3 and we're in the diagram in the question with the White king on c3.








share|improve this answer














share|improve this answer



share|improve this answer








edited 14 hours ago

























answered 15 hours ago









GlorfindelGlorfindel

14.2k45286




14.2k45286












  • $begingroup$
    ... but it doesn't include an explanation why this is the only possible solution (which took me a while to write down...)
    $endgroup$
    – Glorfindel
    14 hours ago






  • 1




    $begingroup$
    @ArnaudMortier Glorfindel's answer eliminates other possibilities.
    $endgroup$
    – noedne
    14 hours ago


















  • $begingroup$
    ... but it doesn't include an explanation why this is the only possible solution (which took me a while to write down...)
    $endgroup$
    – Glorfindel
    14 hours ago






  • 1




    $begingroup$
    @ArnaudMortier Glorfindel's answer eliminates other possibilities.
    $endgroup$
    – noedne
    14 hours ago
















$begingroup$
... but it doesn't include an explanation why this is the only possible solution (which took me a while to write down...)
$endgroup$
– Glorfindel
14 hours ago




$begingroup$
... but it doesn't include an explanation why this is the only possible solution (which took me a while to write down...)
$endgroup$
– Glorfindel
14 hours ago




1




1




$begingroup$
@ArnaudMortier Glorfindel's answer eliminates other possibilities.
$endgroup$
– noedne
14 hours ago




$begingroup$
@ArnaudMortier Glorfindel's answer eliminates other possibilities.
$endgroup$
– noedne
14 hours ago











11












$begingroup$


The last move in this position was Bd5+. White blocked the check with c4, black gave double check by capturing en passant bxc, and the king took the pawn on c3.
chessboard







share|improve this answer









$endgroup$













  • $begingroup$
    Um... to me, it looks like rot13(gur cnja vf ba p2)... :P
    $endgroup$
    – EKons
    14 hours ago










  • $begingroup$
    @EKons I'm not sure I understand what you mean.
    $endgroup$
    – noedne
    14 hours ago










  • $begingroup$
    I've ROT13-ed part of the comment so that it doesn't appear as an immediate spoiler. Check the image you've posted.
    $endgroup$
    – EKons
    14 hours ago












  • $begingroup$
    @EKons I don't see what the problem is.
    $endgroup$
    – noedne
    14 hours ago










  • $begingroup$
    That was super quick. Did you know the problem before? Do you know where it is from?
    $endgroup$
    – Arnaud Mortier
    14 hours ago
















11












$begingroup$


The last move in this position was Bd5+. White blocked the check with c4, black gave double check by capturing en passant bxc, and the king took the pawn on c3.
chessboard







share|improve this answer









$endgroup$













  • $begingroup$
    Um... to me, it looks like rot13(gur cnja vf ba p2)... :P
    $endgroup$
    – EKons
    14 hours ago










  • $begingroup$
    @EKons I'm not sure I understand what you mean.
    $endgroup$
    – noedne
    14 hours ago










  • $begingroup$
    I've ROT13-ed part of the comment so that it doesn't appear as an immediate spoiler. Check the image you've posted.
    $endgroup$
    – EKons
    14 hours ago












  • $begingroup$
    @EKons I don't see what the problem is.
    $endgroup$
    – noedne
    14 hours ago










  • $begingroup$
    That was super quick. Did you know the problem before? Do you know where it is from?
    $endgroup$
    – Arnaud Mortier
    14 hours ago














11












11








11





$begingroup$


The last move in this position was Bd5+. White blocked the check with c4, black gave double check by capturing en passant bxc, and the king took the pawn on c3.
chessboard







share|improve this answer









$endgroup$




The last move in this position was Bd5+. White blocked the check with c4, black gave double check by capturing en passant bxc, and the king took the pawn on c3.
chessboard








share|improve this answer












share|improve this answer



share|improve this answer










answered 15 hours ago









noednenoedne

7,78712261




7,78712261












  • $begingroup$
    Um... to me, it looks like rot13(gur cnja vf ba p2)... :P
    $endgroup$
    – EKons
    14 hours ago










  • $begingroup$
    @EKons I'm not sure I understand what you mean.
    $endgroup$
    – noedne
    14 hours ago










  • $begingroup$
    I've ROT13-ed part of the comment so that it doesn't appear as an immediate spoiler. Check the image you've posted.
    $endgroup$
    – EKons
    14 hours ago












  • $begingroup$
    @EKons I don't see what the problem is.
    $endgroup$
    – noedne
    14 hours ago










  • $begingroup$
    That was super quick. Did you know the problem before? Do you know where it is from?
    $endgroup$
    – Arnaud Mortier
    14 hours ago


















  • $begingroup$
    Um... to me, it looks like rot13(gur cnja vf ba p2)... :P
    $endgroup$
    – EKons
    14 hours ago










  • $begingroup$
    @EKons I'm not sure I understand what you mean.
    $endgroup$
    – noedne
    14 hours ago










  • $begingroup$
    I've ROT13-ed part of the comment so that it doesn't appear as an immediate spoiler. Check the image you've posted.
    $endgroup$
    – EKons
    14 hours ago












  • $begingroup$
    @EKons I don't see what the problem is.
    $endgroup$
    – noedne
    14 hours ago










  • $begingroup$
    That was super quick. Did you know the problem before? Do you know where it is from?
    $endgroup$
    – Arnaud Mortier
    14 hours ago
















$begingroup$
Um... to me, it looks like rot13(gur cnja vf ba p2)... :P
$endgroup$
– EKons
14 hours ago




$begingroup$
Um... to me, it looks like rot13(gur cnja vf ba p2)... :P
$endgroup$
– EKons
14 hours ago












$begingroup$
@EKons I'm not sure I understand what you mean.
$endgroup$
– noedne
14 hours ago




$begingroup$
@EKons I'm not sure I understand what you mean.
$endgroup$
– noedne
14 hours ago












$begingroup$
I've ROT13-ed part of the comment so that it doesn't appear as an immediate spoiler. Check the image you've posted.
$endgroup$
– EKons
14 hours ago






$begingroup$
I've ROT13-ed part of the comment so that it doesn't appear as an immediate spoiler. Check the image you've posted.
$endgroup$
– EKons
14 hours ago














$begingroup$
@EKons I don't see what the problem is.
$endgroup$
– noedne
14 hours ago




$begingroup$
@EKons I don't see what the problem is.
$endgroup$
– noedne
14 hours ago












$begingroup$
That was super quick. Did you know the problem before? Do you know where it is from?
$endgroup$
– Arnaud Mortier
14 hours ago




$begingroup$
That was super quick. Did you know the problem before? Do you know where it is from?
$endgroup$
– Arnaud Mortier
14 hours ago


















draft saved

draft discarded




















































Thanks for contributing an answer to Puzzling Stack Exchange!


  • Please be sure to answer the question. Provide details and share your research!

But avoid



  • Asking for help, clarification, or responding to other answers.

  • Making statements based on opinion; back them up with references or personal experience.


Use MathJax to format equations. MathJax reference.


To learn more, see our tips on writing great answers.




draft saved


draft discarded














StackExchange.ready(
function () {
StackExchange.openid.initPostLogin('.new-post-login', 'https%3a%2f%2fpuzzling.stackexchange.com%2fquestions%2f81057%2fthere-is-only-s%25cc%25b6i%25cc%25b6x%25cc%25b6t%25cc%25b6y-one-place-he-can-be%23new-answer', 'question_page');
}
);

Post as a guest















Required, but never shown





















































Required, but never shown














Required, but never shown












Required, but never shown







Required, but never shown

































Required, but never shown














Required, but never shown












Required, but never shown







Required, but never shown







Popular posts from this blog

He _____ here since 1970 . Answer needed [closed]What does “since he was so high” mean?Meaning of “catch birds for”?How do I ensure “since” takes the meaning I want?“Who cares here” meaningWhat does “right round toward” mean?the time tense (had now been detected)What does the phrase “ring around the roses” mean here?Correct usage of “visited upon”Meaning of “foiled rail sabotage bid”It was the third time I had gone to Rome or It is the third time I had been to Rome

Bunad

Færeyskur hestur Heimild | Tengill | Tilvísanir | LeiðsagnarvalRossið - síða um færeyska hrossið á færeyskuGott ár hjá færeyska hestinum